The balances in two separate bank accounts that grow each month at different rales are represented by the functions f(x) and gix) In what month do the funds in the f(x) bank account exceed those in the glx)
bank account?
Month (x) f(x) = 2* g(x) = 4x + 12
1
2
16
2.
4
20
O Month 3
O Month 4
O Month 5
O Month 6​

Answers

Answer 1

Answer:

The balance in two separate bank accounts grows each month at different rates. the growth rates for both accounts are represented by the functions f(x) = 2x and g(x) = 4x 12. in what month is the f(x) balance greater than the g(x) balance?

Answer:

6 months

Answer 2

function is a relationship between inputs where each input is related to exactly one output.

x = 5,

f(5) = [tex]2^5\\[/tex] = 32

g(5) = 4 x 5 + 12 = 20 + 12 = 32

x = 6,

f(6) = [tex]2^6[/tex] = 64

g(6) = 4 x 6 + 12 = 24 + 12 = 36

At month 6 the funds in the f(x) bank account exceed those in the g(x) bank account.

What is a function?

function is a relationship between inputs where each input is related to exactly one output.

Example:

f(x) = 2x + 1

f(1) = 2 + 1 = 3

f(2) = 2 x 2 + 1 = 4 + 1 = 5

The outputs of the functions are 3 and 5

The inputs of the function are 1 and 2.

We have,

f(x) = [tex]2^{x}[/tex]

g(x) = 4x + 12

x = number of months

Now,

x = 3,

f(3) = 2³ = 8

g(3) = 4 x 3 + 12 = 12 + 12 = 24

x = 4,

f(4) = [tex]2^4[/tex] = 16

g(4) = 4 x 4 + 12 = 16 + 12 = 28

x = 5,

f(5) = [tex]2^5\\[/tex] = 32

g(5) = 4 x 5 + 12 = 20 + 12 = 32

x = 6,

f(6) = [tex]2^6[/tex] = 64

g(6) = 4 x 6 + 12 = 24 + 12 = 36

We see that,

At x = 6,

f(5) = 64

g(5) = 36

Thus,

At month 6 the funds in the f(x) bank account exceed those in the g(x) bank account.

Learn more about functions here:

https://brainly.com/question/28533782

#SPJ2


Related Questions

4. Two unbiased coins are tossed. Calculate the probability that
(a) Two heads are obtained.
(b) One head and one tail is obtained.

Answers

2 coins have four possible outcomes: {HH, HT, TH, TT}

(a) P(2 heads) = 1/4

(b) P(1 head and 1 tail) = 2/4 = 1/2

MY
A circle with radius of 5 cm sits inside a 11 cm x 11 cm rectangle.
Col
What is the area of the shaded region?
Round your final answer to the nearest hundredth.
MY
11 cm
Pro
Pro
Теа
5 cm
11 cm
cm2
2 of 4 OOO


Help

Answers

Step-by-step explanation:

Hi, there!!!

According to the question we must find the area of shaded region, but we must find area of circle and rectangle to find area of shaded region,

So, let's simply work with it,

Firstly, finding the area of rectangle,

length = 11cm.

breadth = 11cm.

now, area= length× breadth.

or, a = 11cm× 11cm.

a= 121cm^2

Now, let's work out the area of circle.

radius= 5cm

and pi. = 3.14 {using pi value as 3.14}

now,

area of a circle = pi× r^2

or, a= 3.14×5^2

or, a = 78.5 cm^2.

Therefore, The area of a circle is 78.5cm^2.

Now lastly finding the area of shadedregion,

area of shaded region = area of rectangle - area of circle.

or, area of shaded region = 121cm^2 - 78.5cm^2

Therefore, the area of shaded region is 42.5 cm^2.

Hope it helps...

Find the distance between the points. Give an exact answer and an approximation to three decimal places.
(3.1,0.3) and (2.7. - 4.9)
The exact distance is
(Simplify your answer. Type an exact ans

Answers

Answer:  sqrt(27.2) =approx 5.215

Step-by-step explanation:

The distance between 2 points can be calculated using Phitagor theorem

L= sqrt( (x1-x2)²+(y1-y2)²)

Where x1, y1 are the coordinates of the first point and  x2, y2 are the coordinates of the 2-nd point.

L=sqrt((3.1-2.7)²+(0.3-(-4.9))²)= sqrt(0.4²+5.2²)=sqrt(27.2) - this is exact answer.

sqrt(27.2)=5.21536...=approx 5.215

Write all the factors of 32
Matj
Answer

Answers

The factors of a number are all the whole integers that can be multiplied to get that number. The factors for 32 are:

1 * 32

2 * 16

4 * 8

1, 2, 4, 8, 16, and 32

Answer: 1, 2, 4, 8, 16, and 32.

Step-by-step explanation:

Factors are what we can multiply to get the number.

Factors of 32:

1 x 32=32

2 x 16=32

4 x 8=32

Therefore, the factors of 32 are 1, 2, 4, 8, 16, and 32.

4. Create your own scenario for the variable expression below. Then, suggest values for the variables and solve. 14x + 12y​

Answers

Answer:

Cost of pencil = $20

Cost of copy = $6

Step-by-step explanation:

Statement.

Gill buys 14 copy and 12 pencils and pays a total $324, if the value of 1 copy and 1 pencil is $26, find cost of copy and pencil.

Computation:

Assume.

Cost of copy = x

Cost of pencil = y

So,

x + y = 26.......Eq1

And

14x + 12y​ = 324.........Eq2

From Eq1 ad Eq2

Cost of pencil = $20

So,

Cost of copy = $6

Niall and Zayn buy 14 concert tickets for them and their friends to go see 5sos and 12 concert tickets for them and their friends to go see Little Mix with a total cost of $648. If the value of 1 5sos ticket and 1 Little Mix ticket is $52, and the Little Mix ticket is $4 more than the 5sos ticket, find cost of both tickets.

5sos = x

Little Mix = y

52 / 2 = 26

26 - 2 = 24

26 + 2 = 26

x = 24

y = 28

5sos tickets = $24 each

Little Mix tickets = $26 each

Please Help Me!!! (WORTH 60 POINTS) Will Give Extra points out

Answers

Answer:

O C.

Step-by-step explanation:

To find a missing side length, use Pythagorean Theorem:

a^2 + b^2 = c^2 where a = 5 b = x and c = 6

5^2 + x^2 = 6^2

25 + x^2 = 36

x^2 = 36 - 25

x^2 = 11

x = 3.3

The value of x is sqrt rt 11

Answer:

√11 cm

Step-by-step explanation:

Pythagorean Thereom

a^2 + b^2= c^2

x^2 +5^2=6^2

x^2 + 25 = 36

subtract 25 from both sides

x^2=11

do the square root

x = √11

Please give me the answer ASAP The average of 5 numbers is 7. If one of the five numbers is removed, the average of the four remaining numbers is 6. What is the value of the number that was removed Show Your Work

Answers

Answer:

The removed number is 11.

Step-by-step explanation:

Given that the average of 5 numbers is 7. So you have to find the total values of 5 numbers :

[tex]let \: x = total \: values[/tex]

[tex] \frac{x}{5} = 7[/tex]

[tex]x = 7 \times 5[/tex]

[tex]x = 35[/tex]

Assuming that the total values of 5 numbers is 35. Next, we have to find the removed number :

[tex]let \: y = removed \: number[/tex]

[tex] \frac{35 - y}{4} = 6[/tex]

[tex]35 - y = 6 \times 4[/tex]

[tex]35 - y = 24[/tex]

[tex]35 - 24 = y[/tex]

[tex]y = 11[/tex]

Okay, let's slightly generalize this

Average of [tex]n[/tex] numbers is [tex]a[/tex]

and then [tex]r[/tex] numbers are removed, and you're asked to find the sum of these [tex]r[/tex] numbers.

Solution:

If average of [tex]n[/tex] numbers is [tex]a[/tex] then the sum of all these numbers is [tex]n\cdot a[/tex]

Now we remove [tex]r[/tex] numbers, so we're left with [tex](n-r)[/tex] numbers. and their. average will be [tex]{\text{sum of these } (n-r) \text{ numbers} \over (n-r)}[/tex] let's call this new average [tex] a^{\prime}[/tex]

For simplicity, say, sum of these [tex]r[/tex] numbers, which are removed is denoted by [tex]x[/tex] .

so the new average is [tex]\frac{\text{Sum of } n \text{ numbers} - x}{n-r}=a^{\prime}[/tex]

or, [tex] \frac{n\cdot a -x}{n-r}=a^{\prime}[/tex]

Simplify the equation, and solve for [tex]x[/tex] to get,

[tex] x= n\cdot a -a^{\prime}(n-r)=n(a-a^{\prime})+ra^{\prime}[/tex]

Hope you understand it :)

7.619 by 10^-3



7.254 by 10^2​

Answers

Answer:

0.007619

0.07254

Step-by-step explanation:

1)7.619*10^-3

0.007619

2)7.254*10^2

0.07254

Explanation:

7.619*10^-3

The number here is 7.619 and the number written in scientific notation has minus 3 as its exponent.

.007.619

So the distance between the first decimal point and the second decimal is only three numbers.

Since it is exponent is minus three.

Another way to get the answer.

[tex]7.619 \times 10 {}^{ - 3} = \frac{7619}{1000} \times \frac{1}{1000} = \frac{7619}{1000000} = 0.007619 [/tex]

This applies to the second one too.

Hope this helps ;) ❤❤❤

You have $2,000 on a credit card that charges a 16% interest rate. If you want to pay off the credit card in 5 years, how much will you need to pay each month (assuming you don't charge anything new to the card)?

Answers

9514 1404 393

Answer:

  $48.64

Step-by-step explanation:

The monthly payment amount is given by the amortization formula ...

  A = P(r/n)/(1 -(1 +r/n)^(-nt))

where P is the loan amount, r is the annual interest rate compounded n times per year for t years.

Here, you have P=2000, r=0.16, n=12 (months per year), t=5 (years), so the payment is ...

  A = $2000(0.16/12)/(1 -(1 +0.16/12)^(-12·5)) = $320/(12(0.54828942))

  A ≈ $48.636 ≈ $48.64

You will need to pay $48.64 each month to pay off the charge in 5 years.

What is the domain of f(x)=2/5x+6​

Answers

Answer:

Look at that picture

Step-by-step explanation:

3. The length of a rectangle is 4 inches more than its width. The area of the rectangle is equal to 5 inches more than 2 times the perimeter. Find the length and width of the rectangle.

Answers

Answer:
Length: sqrt13 + 4
Width: sqrt13
Step-by-step explanation:
Denote the width as x, then the length is x+4. The area of the rectangle will be x(x+4), and the perimeter will then be 4x +8. Hence, you can write the equation x(x+4) - 5 = 4x+8. Solving, you get x = sqrt13.

Solving Equations by Dividing 2) 9x= -135 Solve for x. 0 -144 O 126 O 15 0 -15

Answers

X = 15
Explanation:
9x = -135
Divide by 9 on both sides
X = -15
It is -15 since 135 is negative

[tex]\huge\text{Hey there!}[/tex]

[tex]\huge\boxed{\mathsf{9x = -135}}[/tex]

[tex]\huge\boxed{\text{DIVIDE 9 to BOTH SIDES}}[/tex]

[tex]\huge\boxed{\mathsf{\dfrac{9x}{9}= \dfrac{-135}{9}}}[/tex]

[tex]\huge\boxed{\mathsf{\bullet \ CANCEL: \dfrac{9}{9}\ because\ it \ gives\ you\ 1}}[/tex]

[tex]\huge\boxed{\bullet\ \mathsf{KEEP: \dfrac{-135}{9}\ because\ it\ helps\ solve \ for}}\\\huge\boxed{\mathsf{the\ x-value}}[/tex]

[tex]\huge\boxed{\mathsf{x = \dfrac{-135}{9}}}\\\\\huge\boxed{\mathsf{\dfrac{-135}{9}= x}}}[/tex]

[tex]\huge\boxed{Simplify \ it\uparrow}[/tex]

[tex]\huge\boxed{\mathsf{x = \bf -15}}[/tex]

[tex]\huge\boxed{\textsf{Therefore, your answer is: Option D. -15 }}\huge\checkmark[/tex]

[tex]\huge\text{Good luck on your assignment \& enjoy your day!}[/tex]

~[tex]\huge\boxed{\frak{Amphitrite1040:)}}[/tex]

convert 407 in base 8 to decimal​

Answers

[tex]4\cdot8^2+0\cdot8^1+7\cdot8^0=256+7=263[/tex]

[tex]407_8=263_{10}[/tex]

How many solutions does the following equation have ?
−3x+9−2x=−12−5x

Answers

[tex]\text{Solve for x:}\\\\-3x+9-2x=-12-5x\\\\\text{Combine like terms}\\\\-5x+9=-12-5x\\\\\text{Add 5x to both sides}\\\\9=-12\\\\\text{Since that's not valid, there would be no solutions}\\\\\boxed{\text{No solutions}}[/tex]

Question 2b only! Evaluate using the definition of the definite integral(that means using the limit of a Riemann sum

Answers

Answer:

Hello,

Step-by-step explanation:

We divide the interval [a b] in n equal parts.

[tex]\Delta x=\dfrac{b-a}{n} \\\\x_i=a+\Delta x *i \ for\ i=1\ to\ n\\\\y_i=x_i^2=(a+\Delta x *i)^2=a^2+(\Delta x *i)^2+2*a*\Delta x *i\\\\\\Area\ of\ i^{th} \ rectangle=R(x_i)=\Delta x * y_i\\[/tex]

[tex]\displaystyle \sum_{i=1}^{n} R(x_i)=\dfrac{b-a}{n}*\sum_{i=1}^{n}\ (a^2 +(\dfrac{b-a}{n})^2*i^2+2*a*\dfrac{b-a}{n}*i)\\[/tex]

[tex]=(b-a)^2*a^2+(\dfrac{b-a}{n})^3*\dfrac{n(n+1)(2n+1)}{6} +2*a*(\dfrac{b-a}{n})^2*\dfrac{n (n+1)} {2} \\\\\displaystyle \int\limits^a_b {x^2} \, dx = \lim_{n \to \infty} \sum_{i=1}^{n} R(x_i)\\\\=(b-a)*a^2+\dfrac{(b-a)^3 }{3} +a(b-a)^2\\\\=a^2b-a^3+\dfrac{1}{3} (b^3-3ab^2+3a^2b-a^3)+a^3+ab^2-2a^2b\\\\=\dfrac{b^3}{3}-ab^2+ab^2+a^2b+a^2b-2a^2b-\dfrac{a^3}{3} \\\\\\\boxed{\int\limits^a_b {x^2} \, dx =\dfrac{b^3}{3} -\dfrac{a^3}{3}}\\[/tex]

HELP
PLSFind all the missing elements:

Answers

Answer:

a = 6.7 , c = 2.0

Step-by-step explanation:

For side a

To find the missing side a we use the sine rule

[tex] \frac{ |b| }{ \sin(B) } = \frac{ |a| }{ \sin(A) } [/tex]

From the question

B = 58°

b = 6

A = 109°

Substituting the values into the above formula we have

[tex] \frac{6}{ \sin(58) } = \frac{ |a| }{ \sin(109) } [/tex]

[tex] |a| \sin(58) = 6\sin(109) [/tex]

Divide both sides by sin 58°

[tex] |a| = \frac{6 \sin(108) }{ \sin(58) } [/tex]

a = 6.728791

a = 6.7 to the nearest tenth

For side c

To find side c we use the sine rule

That's

[tex] \frac{ |b| }{ \sin(B) } = \frac{ |c| }{ \sin(C) } [/tex]

C = 13°

[tex] \frac{6}{ \sin(58) } = \frac{ |c| }{ \sin(13) } [/tex]

[tex] |c| \sin(58) = 6 \sin(13) [/tex]

Divide both sides by sin 58°

[tex] |c| = \frac{6 \sin(13) }{ \sin(58) } [/tex]

c = 1.591544

c = 2.0 to the nearest tenth

Hope this helps you

Answer:

B=58 a=6.7 c=1.6

Step-by-step explanation:

It was right on Acellus

Sorry I cant give a better explanation but this unit is killing me.

Decide all proper subsets of A { 8 ,7 ,6 ,5 ,4 ,3 ,2 ,1} = A 1- { 4 ,3 ,2 ,1} 2- { } 3- { 9 ,8 ,7 } 4- { 11 ,2} 5- { 5 }

Answers

Answer:

A, E

Step-by-step explanation:

There should be 2^8-1 proper subsets of A. Its every one besides { }

A car dealer's markup on every car they sell is 20%. For what price did the dealership buy a car that they sold for $18,600?

Answers

Answer:

buying price = $15,500

Step-by-step explanation:

selling price 20% more than the buying price

let the buying price be 100% then;

selling price = 120%

120% = $18,600

100% = ?

(100 × 18600) ÷ 120

= $15,500

Your friend Stacy has given you the following algebraic expression: "Subtract 20
times a number n from twice the cube of the number. What is the expression that your
friend is saying?

Answers

Answer:

Expression = 2n³ - 20n

Step-by-step explanation:

Find:

Expression

Computation:

Assume given number is 'n'

Cube of number = n³

Twice of cube = 2n³

Subtract number = 20n

Expression = 2n³ - 20n

Determine the convergence or divergence of the sequence with the given nth term. If the sequence converges, find its limit. (If the quantity diverges, enter DIVERGES.) an = 1/sqrt(n)

Answers

This sequence converges to 0.

Proof: Recall that

[tex]\displaystyle\lim_{n\to\infty}\frac1{\sqrt n}=0[/tex]

is to say that for any given [tex]\varepsilon>0[/tex], there is some [tex]N[/tex] for which [tex]\left|\frac1{\sqrt n}-0\right|=\frac1{\sqrt n}<\varepsilon[/tex] for all [tex]n>N[/tex].

Let [tex]N=\left\lceil\frac1{\varepsilon^2}\right\rceil[/tex]. Then

[tex]n>\left\lceil\dfrac1{\varepsilon^2}\right\rceil\ge\dfrac1{\varepsilon^2}[/tex]

[tex]\implies\dfrac1n<\varepsilon^2[/tex]

[tex]\implies\dfrac1{\sqrt n}<\varepsilon[/tex]

as required.

PLEASE HELP ASAP THANKS IN ADVANCE

Answers

Answer:

the answer to the question is "C"

Review the example argument and reasoning below. Identify the form (inductive or deductive) of reasoning and the type (example, analogy, causal correlation, syllogism, sign, or causal generalization) of reasoning Raul uses to justify his argument. Then, apply the three tests of argumentative reasoning (quantity, quality, & opposition) to test this argument.

Raul believes that if someone’s eyes shift to the left when they are responding to a question it is evidence that they are lying. While interviewing Michael, Raul notices Michael's eyes shifting to the left frequently when answering questions. Later, Raul tells a coworker that Michael was not hired because Raul believed Michael had lied about his previous experience during the interview.

Answers

Answer:

inductive - . Inductive reasoning makes broad generalizations from specific observations.

casual correlation

quality ( i think)

Step-by-step explanation:

Answer:

A

Step-by-step explanation:

i just did it

Suppose that Y1, Y2,..., Yn denote a random sample of size n from a Poisson distribution with mean λ. Consider λˆ 1 = (Y1 + Y2)/2 and λˆ 2 = Y . Derive the efficiency of λˆ 1 relative to λˆ 2.

Answers

Answer:

The answer is "[tex]\bold{\frac{2}{n}}[/tex]".

Step-by-step explanation:

considering [tex]Y_1, Y_2,........, Y_n[/tex] signify a random Poisson distribution of the sample size of n which means is λ.

[tex]E(Y_i)= \lambda \ \ \ \ \ and \ \ \ \ \ Var(Y_i)= \lambda[/tex]

Let assume that,  

[tex]\hat \lambda_i = \frac{Y_1+Y_2}{2}[/tex]

multiply the above value by Var on both sides:

[tex]Var (\hat \lambda_1 )= Var(\frac{Y_1+Y_2}{2} )[/tex]

            [tex]=\frac{1}{4}(Var (Y_1)+Var (Y_2))\\\\=\frac{1}{4}(\lambda+\lambda)\\\\=\frac{1}{4}( 2\lambda)\\\\=\frac{\lambda}{2}\\[/tex]

now consider [tex]\hat \lambda_2[/tex] = [tex]\bar Y[/tex]

[tex]Var (\hat \lambda_2 )= Var(\bar Y )[/tex]

             [tex]=Var \{ \frac{\sum Y_i}{n}\}[/tex]

             [tex]=\frac{1}{n^2}\{\sum_{i}^{}Var(Y_i)\}\\\\=\frac{1}{n^2}\{ n \lambda \}\\\\=\frac{\lambda }{n}\\[/tex]

For calculating the efficiency divides the [tex]\hat \lambda_1 \ \ \ and \ \ \ \hat \lambda_2[/tex] value:

Formula:

[tex]\bold{Efficiency = \frac{Var(\lambda_2)}{Var(\lambda_1)}}[/tex]

                  [tex]=\frac{\frac{\lambda}{n}}{\frac{\lambda}{2}}\\\\= \frac{\lambda}{n} \times \frac {2} {\lambda}\\\\ \boxed{= \frac{2}{n}}[/tex]

Complete the point-slope equation of the line through (2,3)(7,4). Use exact numbers. y-4=


Please help me, I would really appreciate it!

Answers

Answer:

The answer is

[tex]y - 4 = \frac{1}{5} (x - 7)[/tex]

Step-by-step explanation:

To find the equation of a line given two points first find the slope and use the formula

[tex] y - y_{1} = m(x - x_{1})[/tex]

Where m is the slope

To find the slope we use the formula

[tex]m = \frac{y2 - y1}{x2 - x1} [/tex]

The slope of the line using points

(2,3)(7,4) is

[tex]m = \frac{4 - 3}{7 - 2} = \frac{1}{5} [/tex]

Equation of the line using point (7,4) and slope 1/5 is

[tex]y - 4 = \frac{1}{5} (x - 7)[/tex]

Hope this helps you

Answer:

y-4=1/5(x-3)

Step-by-step explanation:

We plug in the x's and the y's and find the slope with:

[tex](y-y_{1} )/ x-x_{1})=m[/tex]

Please answer this correctly without making mistakes

Answers

Answer:

[tex]\large \boxed{\mathrm{4/5 \ cups}}[/tex]

Step-by-step explanation:

Subtract 1/10 from 9/10 to find out how much is left.

9/10 - 1/10

8/10 = 4/5

Answer:

4/5 cups

Step-by-step explanation:

[tex]Volume\:of \: syrup \:in \:cup\:from\:jug = \frac{9}{10}\\\\ She \:took\: \frac{1}{10} from \:the\:cup\:into\:the \:jug \\\\Volume \:of syrup\:in\:cup=?\\\\\frac{9}{10} -\frac{1}{10} \\\\= \frac{4}{5} cups[/tex]

3x7-12-2=7?


I have no clue please help...

Answers

Answer:

X = 28/3, or 9 1/3 or 9.3

Step-by-step explanation:

Answer:

Step-by-step explanation:


Find the solution of the system of equations.
2x – 10y = -28
-10x + 10y = -20
GbA

Answers

Answer:

(6, 4 )

Step-by-step explanation:

Given the 2 equations

2x - 10y = - 28 → (1)

- 10x + 10y = - 20 → (2)

Adding (1) and (2) term by term eliminates the term in y, that is

- 8x = - 48 ( divide both sides by - 8 )

x = 6

Substitute x = 6 into either of the 2 equations and evaluate for y

Substituting into (1)

2(6) - 10y = - 28

12 - 10y = - 28 ( subtract 12 from both sides )

- 10y = - 40 ( divide both sides by - 10 )

y = 4

Solution is (6, 4 )

NO LINKS OR ANSWERING QUESTIONS YOU DON'T KNOW!!!

1. How can a matrix be used to solve a system of equations? Demonstrate by solving the following system. Show your work. In other words, use a problem of system of equations problem as an example.

Answers

Answer:

Step-by-step explanation:

Assuming the system is solvable in the first place, create an augmented matrix of coefficients from the equations. Then put the matrix into reduced row echelon form.

Example is attached.

"Demonstrate by solving the following system."

You need to provide the system of equations.

I need help plotting this on number line

Answers

Answer:

Step-by-step explanation:

Yooo I just had MAD diarrhea D:

Answers

I’m sorry I usually drink water you can lose a lot of water while doing that . Try to not a lot maybe some crackers
Other Questions
12. Consider the function (x) = x^4 x^3 + 2x^2 2x. How many real roots does it have? options: A) 2 B) 1 C) 3 D) 4 15. Karla Salons leased equipment from Smith Co. on July 1, 2021, in a finance lease. The present value of the lease payments discounted at 10% was $81,100. Ten annual lease payments of $12,000 are due each year beginning July 1, 2021. Smith Co. had constructed the equipment recently for $66,000, and its retail fair value was $81,100. What amount of interest revenue from the lease should Smith Co. report in its December 31, 2021, income statement The generator in a purely inductive AC circuit has an angular frequency of 363 rad/s. If the maximum voltage is 169 V and the inductance is 0.0937 H, what is the rms current in the circuit Decide whether the sentence is grammatically CORRECT or INCORRECT as written.T asististe a la escuela ayer. (asistir)correctincorrect $10,000 for 20 years at 5% compounded annually A survey was taken of children between the ages of 3 and 7. Let A be the event that the person has 2 siblings, and let B be the event that the person does not have a pet.Which statement is true about whether A and B are independent events?A and B are independent events because P(AB) = P(A) = 0.18.A and B are independent events because P(AB) = P(A) = 0.4.A and B are not independent events because P(AB) = 0.4 and P(A) = 0.18.A and B are not independent events because P(AB) = 0.18 and P(A) = 0.4. 6. How are excess salts that accumulate in cells transferred to the blood stream so they can be removed from the body? Explain how this process works in terms of tonicity. lets try this again plz help If an economist wishes to determine whether there is evidence that average family incomes in a community exceeds $25,000:_______a. either a one-tailed or two-tailed test could be used with equivalent results. b. a one-tailed test should be utilized. c. a two-tailed test should be utilized. d. None of the above. A hollow metal sphere has an external diameter of 12 cm and a thickness of 2 cm. (i) Given that the mass of 1 cm of the metal is 5.4 g, find the mass of the hollow sphere in kg. (ii) The hollow sphere is melted and recast to form a solid sphere. Find the radius of the solid sphere. Which group would advertisers want to target and with what type of advertisement immediately before a holiday, as opposed to during a non-holiday time Sailor SheftatAngle Terminology with EquationsAug 03, 8:01:46 AM?ZA and B are complementary angles. If mZA= (6x 4) and mZB = (x + 17), then find the measure of ZA.BEAnswer:Submit Answerattempt 1 out of 2Pls help PLS HELP ME ON THIS QUESTION I WILL MARK YOU AS BRAINLIEST IF YOU KNOW THE ANSWER PLS GIVE ME A STEP BY STEP EXPLANATION!! Express the product of z1 and z2 in standard form given that [tex]z_{1} = -3[cos(\frac{-\pi }{4} )+isin(\frac{-\pi }{4} )][/tex] and [tex]z_{2} = 2\sqrt{2} [cos(\frac{-\pi }{2} )+isin(\frac{-\pi }{2} )][/tex] Assume that a U.S. firm considers investing in British one-year Treasury securities. The interest rate on these securities is 12%, while the interest rate on the same securities in the U.S. is 10%. The firm believes that today's spot rate is an appropriate forecast for the spot rate of the pound in one year. Based on this information, the effective yield on British securities from the U.S. firm's perspective is: WILL GIVE BRAINLIEST!!!!!!!!!!!!!!!!! In the following graph, ABC is congruent to ABC. A teacher asks Janine to state the translation rule for this transformation. She focuses on point C and states that the translation rule is (x,y)(x-2, y-6). In other words, to shift from the blue to the red triangle, you must shift each point two units to the left and six units down. Janine made an error. Explain how to correct her rule by either using transformation notation showing each step or explain using 2-3 sentences. Keisha has completed 72 math problems in her SHSAT practice workbook. This is 24% of the total number of math problems in the workbook. How many math problems are in the workbook? what did hilter did to the German I need help on this :( HELP ASAP The figure shows two parallel lines AB and DE cut by the transversals AE and BD. Which best explains the relationship between triangle ABC and triangle EDC?